Can Linear Independence Affect Pivot Positions in Matrix Columns?

Click For Summary
The discussion centers on proving that if a vector v in Rn is not a linear combination of the columns of an n x n matrix A, then at least one column of A is not a pivot column. The user outlines their reasoning, linking the absence of v as a linear combination to the lack of pivot positions in the reduced row echelon form (rref) of A. They express uncertainty about transitioning between theorems regarding pivot positions and linear combinations, specifically questioning the applicability of theorem 4 (for m x n matrices) to theorem 8 (for n x n matrices). The conversation highlights the importance of understanding the relationship between pivot positions and the span of matrix columns in linear algebra. Clarifying these concepts is essential for constructing a valid proof.
Cali210
Messages
1
Reaction score
0

Homework Statement



So the question is, Prove the following:

Let A be an n x n matrix. If there exists a vector v in Rn that is not a linear
combination of the columns of A, then at least one column of A is not a pivot column.

Homework Equations



The only relevant theorem I think is the invertible matrix theorem, which i attached.
I also attached theorem 4 (book has different names)

The Attempt at a Solution



So far, I started with

- Let A be a n x n matrix and v be a vector in Rn that is not a linear combination of the columns of A
- then there is not a pivot position in every row of rref A (theorem 8 not g to not c)
- then there is at most n-1 pivot positions (out of n rows)
- then at least one column of rref A is not a pivot position (square matrix).

the question i have is, am i allowed to jump from my first sentence to my second sentence without any justification?

For some reason, I am thinking that I'm supposed to go from (theorem 4 not b to not a)
and then, since theorem 4a is equivalent to theorem 8g, then jump from (theorem 8 not g to not c). But am i allowed to use and jump from theorem 4 to theorem 8? since theorem 4 is for a m x n matrix, while theorem 8 is n x n matrix?

Also, in theorem 4 there's a statement " each b in Rm is a linear combination of the columns of A", which is the assumption i started with. What would be the equivalent statement in theorem 8, if there is any?

Sorry, this is probably a basic proof question, but I'm just horrible at proving something, so I wanted to make sure I was on the right path.
 

Attachments

  • 103thm2.8.jpg
    103thm2.8.jpg
    29.3 KB · Views: 474
  • 103thm1.4.4.jpg
    103thm1.4.4.jpg
    13.7 KB · Views: 467
Last edited:
Physics news on Phys.org
What about span?
 
Question: A clock's minute hand has length 4 and its hour hand has length 3. What is the distance between the tips at the moment when it is increasing most rapidly?(Putnam Exam Question) Answer: Making assumption that both the hands moves at constant angular velocities, the answer is ## \sqrt{7} .## But don't you think this assumption is somewhat doubtful and wrong?

Similar threads

  • · Replies 4 ·
Replies
4
Views
2K
  • · Replies 3 ·
Replies
3
Views
4K
  • · Replies 2 ·
Replies
2
Views
2K
  • · Replies 15 ·
Replies
15
Views
3K
  • · Replies 3 ·
Replies
3
Views
3K
  • · Replies 3 ·
Replies
3
Views
1K
  • · Replies 2 ·
Replies
2
Views
2K
  • · Replies 4 ·
Replies
4
Views
2K
  • · Replies 5 ·
Replies
5
Views
1K
Replies
9
Views
2K